+1 Daumen
1,7k Aufrufe

Hallo an alle!

Ich habe auch den Weg hierhin gefunden... und habe eine Frage:

Hat die Gleichung w²+x²+y²+z²=2wxyz außer (0,0,0,0) noch ganzzahlige Lösungen?

Ich selber habe keine weiteren gefunden... könnt ihr mir helfen?



Klaus

Avatar von
Gute Frage: Gemäss Rubrik Integer in

https://www.wolframalpha.com/input/?i=w²%2Bx²%2By²%2Bz²%3D2wxyz

gibts keine weiteren ganzzahligen Lösungen.


Aber wie beweise ich das... ?

3 Antworten

+1 Daumen
 
Beste Antwort

Man kann zuerst mit der Teilbarkeit durch 4 zeigen, dass w,x,y,z gerade sein müssen (m,n,p,q seien ganze Zahlen):

Wenn ich eine ungerade Zahl quadriere, erhalte ich eine Zahl, die beim Teilen durch 4 immer Rest 1 ergibt:

(2n+1)2 = 4(n2+n) + 1

- alle vier können nicht ungerade sein, weil sonst die linke Seite durch 4 teilbar wäre, während die rechte Seite einen Rest von 2 ergibt, teilt man sie durch 4:

(2n+1)4 = 2m+1 ist ungerade, 2*(2m+1)= 4m + 2

- drei ungerade, eine gerade  Lösungen: dann hätten wir links eine ungerade Summe, rechts eine gerade Zahl wegen des Faktors 2 und wegen der geraden Lösung

- zwei ungerade, zwei gerade Lösungen: links: 4(n2+n) + 1 + 4(m2+m) + 1 + 4p + 4q ergibt Rest 2, 2wxyz ist durch 4 teilbar wegen des Faktors 2 und wegen der geraden Lösungen

- eine ungerade, drei gerade: dann hätten wir links eine ungerade Summe, rechts eine gerade Zahl wegen des Faktors 2 und wegen den 3 geraen Lösungen

- es bleibt nur noch die Möglichkeit, dass alle 4 Lösungen gerade sind. Dann haben wir rechts entweder 32 oder ein Vielfaches davon: 2wxyz = 2*(2m)*(2n)*(2p)*(2q) =2^5*mnpq = 32*mnpq

Es enthalten alle Lösungen den Faktor 2. Ich kann jetzt eine Substitution machen, indem ich alle durch 2 teile:

w1 sei w/2, x1 sei x/2, y1 sei y/2, z1 sei z/2. Das führt uns wieder zur ursprünglichen Gleichung

w12+x12+y12+z12= 2*w1x1y1z1. Auch hier müssten alle Lösungen wieder gerade sein. Wenn ich jetzt wieder die Substitution mache und das unendlich mal wiederhole, folgt daraus, dass alle Lösungen w,x,y,z den Faktor 2unendlich haben müssten. Daraus folgt: (0;0;0;0) ist die einzige mögliche Lösung.

 

Avatar von 2,3 k

Sehr schön herausgearbeitet!

Erinnert mich übrigens an: Warum ergibt eine gerade Zahl quadriert auch eine gerade Zahl?

Super, danke!

Ich glaub, das hab ich jetzt auch verstanden :) hätt ich nicht gedacht, sah anfangs so kompliziert aus... aber wirklich toll!
0 Daumen
Wenn eine Ziffer Null ist ist die Gleichung nicht erfüllt. Also müssen alles ganze Zahlen <> 0 sein.

Da auf der linken Seite der Term Positiv ist muss der Rechts auch positiv sein. Damit kommen auch keine negativen Zahlen in Frage.

Die kleinsten ganzen Zahlen 1 führen Links aber schon zu einem zu großen Wert. Daher gibt es keine ganzzahligen Lösungen außer die Triviallösung, dass alle Variablen Null sind.
Avatar von 477 k 🚀
Aber das ist ja noch kein richtiger Beweis... oder?
Da ist auch ein Denkfehler drin.
Na wenn du sagst ;)

Kann jemand anders mir weiterhelfen?
Hallo :)

Wollte mal kurz hier an die Frage erinnern... die Frage ist noch offen! Danke :)
0 Daumen

Mal ein Ansatz:

 

Du kannst du Gleichung zu w²-(2xyz)w+x²+y²+z²=0 umformen. Hast du dann eine Lösung (w0,x0,y0,z0), so ist nach dem Satz des Vieta auch (2x0y0z0-w0,x0,y0,z0) eine Lösung. Hast du also eine Lösung, kannst du dir unendlich viele "generieren". Das geht aber bei der Lösung (0,0,0,0) nicht.

 

Vielleicht hilft das weiter.

Avatar von


danke für die Antwort.

 

Aber wir wissen ja schon, dass es keine weiteren Lösungen gibt. Dann kann man deins ja noch so verwenden, dass man sagt:

Angenommen man hätte ein weitere Lösung außer (0,0,0,0). Man nehme die, mit der kleinsten Summe w+x+y+z. Mithilfe deiner Formel zeigt man dann, dass es eine weitere Lösung geben muss, bei der die Summe kleiner ist, als die angenommene kleinste.

Das wäre dann ein Beweis durch Widerspruch.

 

Aber leider fehlt da ja noch einiges...

Ein anderes Problem?

Stell deine Frage

Willkommen bei der Mathelounge! Stell deine Frage einfach und kostenlos

x
Made by a lovely community